ChaseDream

标题: GWD-23-Q39 [打印本页]

作者: 2006gmat2006    时间: 2006-10-31 07:54
标题: GWD-23-Q39

1.       

In Gandania, where the government has a monopoly on tobacco sales, the incidence of smoking-related health problems has risen steadily for the last twenty years.  The health secretary recently proposed a series of laws aimed at curtailing tobacco use in Gandania.  Profits from tobacco sales, however, account for ten percent of Gandania’s annual revenues.  Therefore, Gandania cannot afford to institute the proposed laws.

 

Which of the following, if true, most seriously weakens the argument?

 

  1. All health care in Gandania is government-funded.

  2. Implementing the proposed laws is not likely to cause a significant increase in the amount of tobacco Gandania exports.

  3. The percentage of revenue Gandania receives from tobacco sales has remained steady in recent years.

  4. Profits from tobacco sales far surpass any other single source of revenue for the Gandanian government.

  5. No government official in Gandania has ever previously proposed laws aimed at curtailing tobacco use.

Why the answer is A?!


作者: gonghao    时间: 2006-10-31 21:09

国家卖烟草

国家公费医疗

此高彼低

因此weaken


作者: 2006gmat2006    时间: 2006-11-1 02:16

thanks a lot!


作者: heshaohua666    时间: 2006-12-12 00:21

啊,你牛啊


作者: Koror    时间: 2006-12-12 12:09
Why C is rong?
作者: xiaoda    时间: 2008-7-2 16:25
up




欢迎光临 ChaseDream (https://forum.chasedream.com/) Powered by Discuz! X3.3